Uniformly continuous derivative implies existence of limitHow to show that a uniformly continuous function is bounded?Simple Construction of a Uniformly Continuous Real Valued Function With No Derivative Anywhere In The Domain?Bounded derivative implies uniform continuity- does the domain need to be an open interval?Prove $f$ is uniformly continuous iff $ lim_xto inftyf(x)=0$The product of uniformly continuous functions is not necessarily uniformly continuousIs $f$ uniformly continuous?Continuous function goes to zero at $pm infty$, show it is uniformly continuousDifficult limit problem involving sine and tangent$f$ is uniformly continuous if and only if the limits exist in $mathbbR$Relationship with uniformly continuous function and its derivative.

Packing rectangles: Does rotation ever help?

Realistic Necromancy?

Does a semiconductor follow Ohm's law?

Does Gita support doctrine of eternal samsara?

What does KSP mean?

What's the polite way to say "I need to urinate"?

Any examples of headwear for races with animal ears?

How did Captain America manage to do this?

Uniformly continuous derivative implies existence of limit

Sci-fi novel series with instant travel between planets through gates. A river runs through the gates

Please, smoke with good manners

How to have a sharp product image?

How to creep the reader out with what seems like a normal person?

Binary Numbers Magic Trick

Who is the Umpire in this picture?

Mac Pro install disk keeps ejecting itself

Combinable filters

What are the potential pitfalls when using metals as a currency?

With a Canadian student visa, can I spend a night at Vancouver before continuing to Toronto?

The Defining Moment

Term for maladaptive animal behavior that will lead to their demise?

simple conditions equation

how to sum variables from file in bash

Why isn't the definition of absolute value applied when squaring a radical containing a variable?



Uniformly continuous derivative implies existence of limit


How to show that a uniformly continuous function is bounded?Simple Construction of a Uniformly Continuous Real Valued Function With No Derivative Anywhere In The Domain?Bounded derivative implies uniform continuity- does the domain need to be an open interval?Prove $f$ is uniformly continuous iff $ lim_xto inftyf(x)=0$The product of uniformly continuous functions is not necessarily uniformly continuousIs $f$ uniformly continuous?Continuous function goes to zero at $pm infty$, show it is uniformly continuousDifficult limit problem involving sine and tangent$f$ is uniformly continuous if and only if the limits exist in $mathbbR$Relationship with uniformly continuous function and its derivative.













2












$begingroup$



Let $f in C^1([0, +infty))$. Suppose that $lim_x rightarrow +infty f(x)=L$ and $f'$ is uniformly continuous.



Show that $$lim_x rightarrow +infty f'(x) + f(x)=L$$




I tried to apply L'Hospital's Rule to $frace^xf(x)e^x$ since $fracddxe^xf(x)=e^x(f'(x)+f(x))$. It seems alright but I didn't use the uniform continuity of $f'$ and it doesn't work for the function $f(x)=fracsin(x^2)x$ whose derivative is $f'(x)=2cos(x^2)-fracsin(x^2)x^2$ since $lim_x rightarrow +infty f'(x)$ doesn't exist.



Any ideas? Thanks in advance.










share|cite|improve this question









$endgroup$











  • $begingroup$
    The L'Hospital trick won't work in cases where the limit of $[f(x) +f'(x)]$ does not exist as in your example.
    $endgroup$
    – RRL
    1 hour ago















2












$begingroup$



Let $f in C^1([0, +infty))$. Suppose that $lim_x rightarrow +infty f(x)=L$ and $f'$ is uniformly continuous.



Show that $$lim_x rightarrow +infty f'(x) + f(x)=L$$




I tried to apply L'Hospital's Rule to $frace^xf(x)e^x$ since $fracddxe^xf(x)=e^x(f'(x)+f(x))$. It seems alright but I didn't use the uniform continuity of $f'$ and it doesn't work for the function $f(x)=fracsin(x^2)x$ whose derivative is $f'(x)=2cos(x^2)-fracsin(x^2)x^2$ since $lim_x rightarrow +infty f'(x)$ doesn't exist.



Any ideas? Thanks in advance.










share|cite|improve this question









$endgroup$











  • $begingroup$
    The L'Hospital trick won't work in cases where the limit of $[f(x) +f'(x)]$ does not exist as in your example.
    $endgroup$
    – RRL
    1 hour ago













2












2








2


1



$begingroup$



Let $f in C^1([0, +infty))$. Suppose that $lim_x rightarrow +infty f(x)=L$ and $f'$ is uniformly continuous.



Show that $$lim_x rightarrow +infty f'(x) + f(x)=L$$




I tried to apply L'Hospital's Rule to $frace^xf(x)e^x$ since $fracddxe^xf(x)=e^x(f'(x)+f(x))$. It seems alright but I didn't use the uniform continuity of $f'$ and it doesn't work for the function $f(x)=fracsin(x^2)x$ whose derivative is $f'(x)=2cos(x^2)-fracsin(x^2)x^2$ since $lim_x rightarrow +infty f'(x)$ doesn't exist.



Any ideas? Thanks in advance.










share|cite|improve this question









$endgroup$





Let $f in C^1([0, +infty))$. Suppose that $lim_x rightarrow +infty f(x)=L$ and $f'$ is uniformly continuous.



Show that $$lim_x rightarrow +infty f'(x) + f(x)=L$$




I tried to apply L'Hospital's Rule to $frace^xf(x)e^x$ since $fracddxe^xf(x)=e^x(f'(x)+f(x))$. It seems alright but I didn't use the uniform continuity of $f'$ and it doesn't work for the function $f(x)=fracsin(x^2)x$ whose derivative is $f'(x)=2cos(x^2)-fracsin(x^2)x^2$ since $lim_x rightarrow +infty f'(x)$ doesn't exist.



Any ideas? Thanks in advance.







real-analysis






share|cite|improve this question













share|cite|improve this question











share|cite|improve this question




share|cite|improve this question










asked 2 hours ago









lzralbulzralbu

697512




697512











  • $begingroup$
    The L'Hospital trick won't work in cases where the limit of $[f(x) +f'(x)]$ does not exist as in your example.
    $endgroup$
    – RRL
    1 hour ago
















  • $begingroup$
    The L'Hospital trick won't work in cases where the limit of $[f(x) +f'(x)]$ does not exist as in your example.
    $endgroup$
    – RRL
    1 hour ago















$begingroup$
The L'Hospital trick won't work in cases where the limit of $[f(x) +f'(x)]$ does not exist as in your example.
$endgroup$
– RRL
1 hour ago




$begingroup$
The L'Hospital trick won't work in cases where the limit of $[f(x) +f'(x)]$ does not exist as in your example.
$endgroup$
– RRL
1 hour ago










1 Answer
1






active

oldest

votes


















3












$begingroup$

We have $lim_x to infty f'(x) = 0$ because,



$$int_0^x f'(t) , dt = f(x) - f(0), \int_0^infty f'(t) , dt = lim_x to inftyf(x) - f(0) = L - f(0) quad (textconvergent)$$



and $f'$ is uniformly continuous.



To prove this assume that $lim_x to inftyf'(x) =0$ does not hold and arrive at contradiction with the fact that the integral of $f'$ is convergent.



If $lim_x to infty f'(x) = 0$ does not hold then there exists $epsilon_0 > 0$ and a sequence $x_n to infty$ such that $|f'(x_n)| geqslant epsilon_0$ for all $n$. Next apply uniform continuity.



Assume WLOG that $f'(x_n) geqslant epsilon_0$.



There exists by uniform continuity $delta > 0$ such that $|f'(t) - f'(x_n)| < epsilon_0/2 implies f'(t) > epsilon_0/2$ for all $t in [x_n - delta,x_n + delta],$ and



$$ int_x_n - delta^x_n + delta f'(t) , dt > epsilondelta$$



This violates the Cauchy criterion for convergence of the improper integral since $x_n$ can be arbitrarily large.






share|cite|improve this answer











$endgroup$












  • $begingroup$
    I can help you further, but first let me know if these hints makes it obvious to you now.
    $endgroup$
    – RRL
    1 hour ago










  • $begingroup$
    I still can't see how to use uniform continuity. Could you, please, explain it further?
    $endgroup$
    – lzralbu
    1 hour ago










  • $begingroup$
    I shall do so...
    $endgroup$
    – RRL
    1 hour ago










  • $begingroup$
    What about the example given in the question?
    $endgroup$
    – Jens Schwaiger
    46 mins ago










  • $begingroup$
    @JensSchwaiger: $cos(x^2)$ is not uniformly continuous on $[0,infty)$. OP introduced this as a counterexample for the L'Hospital trick. It is not relevant to the actual question where the assumption is that $f'$ is uniformly continuous.
    $endgroup$
    – RRL
    32 mins ago











Your Answer








StackExchange.ready(function()
var channelOptions =
tags: "".split(" "),
id: "69"
;
initTagRenderer("".split(" "), "".split(" "), channelOptions);

StackExchange.using("externalEditor", function()
// Have to fire editor after snippets, if snippets enabled
if (StackExchange.settings.snippets.snippetsEnabled)
StackExchange.using("snippets", function()
createEditor();
);

else
createEditor();

);

function createEditor()
StackExchange.prepareEditor(
heartbeatType: 'answer',
autoActivateHeartbeat: false,
convertImagesToLinks: true,
noModals: true,
showLowRepImageUploadWarning: true,
reputationToPostImages: 10,
bindNavPrevention: true,
postfix: "",
imageUploader:
brandingHtml: "Powered by u003ca class="icon-imgur-white" href="https://imgur.com/"u003eu003c/au003e",
contentPolicyHtml: "User contributions licensed under u003ca href="https://creativecommons.org/licenses/by-sa/3.0/"u003ecc by-sa 3.0 with attribution requiredu003c/au003e u003ca href="https://stackoverflow.com/legal/content-policy"u003e(content policy)u003c/au003e",
allowUrls: true
,
noCode: true, onDemand: true,
discardSelector: ".discard-answer"
,immediatelyShowMarkdownHelp:true
);



);













draft saved

draft discarded


















StackExchange.ready(
function ()
StackExchange.openid.initPostLogin('.new-post-login', 'https%3a%2f%2fmath.stackexchange.com%2fquestions%2f3205125%2funiformly-continuous-derivative-implies-existence-of-limit%23new-answer', 'question_page');

);

Post as a guest















Required, but never shown

























1 Answer
1






active

oldest

votes








1 Answer
1






active

oldest

votes









active

oldest

votes






active

oldest

votes









3












$begingroup$

We have $lim_x to infty f'(x) = 0$ because,



$$int_0^x f'(t) , dt = f(x) - f(0), \int_0^infty f'(t) , dt = lim_x to inftyf(x) - f(0) = L - f(0) quad (textconvergent)$$



and $f'$ is uniformly continuous.



To prove this assume that $lim_x to inftyf'(x) =0$ does not hold and arrive at contradiction with the fact that the integral of $f'$ is convergent.



If $lim_x to infty f'(x) = 0$ does not hold then there exists $epsilon_0 > 0$ and a sequence $x_n to infty$ such that $|f'(x_n)| geqslant epsilon_0$ for all $n$. Next apply uniform continuity.



Assume WLOG that $f'(x_n) geqslant epsilon_0$.



There exists by uniform continuity $delta > 0$ such that $|f'(t) - f'(x_n)| < epsilon_0/2 implies f'(t) > epsilon_0/2$ for all $t in [x_n - delta,x_n + delta],$ and



$$ int_x_n - delta^x_n + delta f'(t) , dt > epsilondelta$$



This violates the Cauchy criterion for convergence of the improper integral since $x_n$ can be arbitrarily large.






share|cite|improve this answer











$endgroup$












  • $begingroup$
    I can help you further, but first let me know if these hints makes it obvious to you now.
    $endgroup$
    – RRL
    1 hour ago










  • $begingroup$
    I still can't see how to use uniform continuity. Could you, please, explain it further?
    $endgroup$
    – lzralbu
    1 hour ago










  • $begingroup$
    I shall do so...
    $endgroup$
    – RRL
    1 hour ago










  • $begingroup$
    What about the example given in the question?
    $endgroup$
    – Jens Schwaiger
    46 mins ago










  • $begingroup$
    @JensSchwaiger: $cos(x^2)$ is not uniformly continuous on $[0,infty)$. OP introduced this as a counterexample for the L'Hospital trick. It is not relevant to the actual question where the assumption is that $f'$ is uniformly continuous.
    $endgroup$
    – RRL
    32 mins ago















3












$begingroup$

We have $lim_x to infty f'(x) = 0$ because,



$$int_0^x f'(t) , dt = f(x) - f(0), \int_0^infty f'(t) , dt = lim_x to inftyf(x) - f(0) = L - f(0) quad (textconvergent)$$



and $f'$ is uniformly continuous.



To prove this assume that $lim_x to inftyf'(x) =0$ does not hold and arrive at contradiction with the fact that the integral of $f'$ is convergent.



If $lim_x to infty f'(x) = 0$ does not hold then there exists $epsilon_0 > 0$ and a sequence $x_n to infty$ such that $|f'(x_n)| geqslant epsilon_0$ for all $n$. Next apply uniform continuity.



Assume WLOG that $f'(x_n) geqslant epsilon_0$.



There exists by uniform continuity $delta > 0$ such that $|f'(t) - f'(x_n)| < epsilon_0/2 implies f'(t) > epsilon_0/2$ for all $t in [x_n - delta,x_n + delta],$ and



$$ int_x_n - delta^x_n + delta f'(t) , dt > epsilondelta$$



This violates the Cauchy criterion for convergence of the improper integral since $x_n$ can be arbitrarily large.






share|cite|improve this answer











$endgroup$












  • $begingroup$
    I can help you further, but first let me know if these hints makes it obvious to you now.
    $endgroup$
    – RRL
    1 hour ago










  • $begingroup$
    I still can't see how to use uniform continuity. Could you, please, explain it further?
    $endgroup$
    – lzralbu
    1 hour ago










  • $begingroup$
    I shall do so...
    $endgroup$
    – RRL
    1 hour ago










  • $begingroup$
    What about the example given in the question?
    $endgroup$
    – Jens Schwaiger
    46 mins ago










  • $begingroup$
    @JensSchwaiger: $cos(x^2)$ is not uniformly continuous on $[0,infty)$. OP introduced this as a counterexample for the L'Hospital trick. It is not relevant to the actual question where the assumption is that $f'$ is uniformly continuous.
    $endgroup$
    – RRL
    32 mins ago













3












3








3





$begingroup$

We have $lim_x to infty f'(x) = 0$ because,



$$int_0^x f'(t) , dt = f(x) - f(0), \int_0^infty f'(t) , dt = lim_x to inftyf(x) - f(0) = L - f(0) quad (textconvergent)$$



and $f'$ is uniformly continuous.



To prove this assume that $lim_x to inftyf'(x) =0$ does not hold and arrive at contradiction with the fact that the integral of $f'$ is convergent.



If $lim_x to infty f'(x) = 0$ does not hold then there exists $epsilon_0 > 0$ and a sequence $x_n to infty$ such that $|f'(x_n)| geqslant epsilon_0$ for all $n$. Next apply uniform continuity.



Assume WLOG that $f'(x_n) geqslant epsilon_0$.



There exists by uniform continuity $delta > 0$ such that $|f'(t) - f'(x_n)| < epsilon_0/2 implies f'(t) > epsilon_0/2$ for all $t in [x_n - delta,x_n + delta],$ and



$$ int_x_n - delta^x_n + delta f'(t) , dt > epsilondelta$$



This violates the Cauchy criterion for convergence of the improper integral since $x_n$ can be arbitrarily large.






share|cite|improve this answer











$endgroup$



We have $lim_x to infty f'(x) = 0$ because,



$$int_0^x f'(t) , dt = f(x) - f(0), \int_0^infty f'(t) , dt = lim_x to inftyf(x) - f(0) = L - f(0) quad (textconvergent)$$



and $f'$ is uniformly continuous.



To prove this assume that $lim_x to inftyf'(x) =0$ does not hold and arrive at contradiction with the fact that the integral of $f'$ is convergent.



If $lim_x to infty f'(x) = 0$ does not hold then there exists $epsilon_0 > 0$ and a sequence $x_n to infty$ such that $|f'(x_n)| geqslant epsilon_0$ for all $n$. Next apply uniform continuity.



Assume WLOG that $f'(x_n) geqslant epsilon_0$.



There exists by uniform continuity $delta > 0$ such that $|f'(t) - f'(x_n)| < epsilon_0/2 implies f'(t) > epsilon_0/2$ for all $t in [x_n - delta,x_n + delta],$ and



$$ int_x_n - delta^x_n + delta f'(t) , dt > epsilondelta$$



This violates the Cauchy criterion for convergence of the improper integral since $x_n$ can be arbitrarily large.







share|cite|improve this answer














share|cite|improve this answer



share|cite|improve this answer








edited 1 hour ago

























answered 1 hour ago









RRLRRL

54.1k52675




54.1k52675











  • $begingroup$
    I can help you further, but first let me know if these hints makes it obvious to you now.
    $endgroup$
    – RRL
    1 hour ago










  • $begingroup$
    I still can't see how to use uniform continuity. Could you, please, explain it further?
    $endgroup$
    – lzralbu
    1 hour ago










  • $begingroup$
    I shall do so...
    $endgroup$
    – RRL
    1 hour ago










  • $begingroup$
    What about the example given in the question?
    $endgroup$
    – Jens Schwaiger
    46 mins ago










  • $begingroup$
    @JensSchwaiger: $cos(x^2)$ is not uniformly continuous on $[0,infty)$. OP introduced this as a counterexample for the L'Hospital trick. It is not relevant to the actual question where the assumption is that $f'$ is uniformly continuous.
    $endgroup$
    – RRL
    32 mins ago
















  • $begingroup$
    I can help you further, but first let me know if these hints makes it obvious to you now.
    $endgroup$
    – RRL
    1 hour ago










  • $begingroup$
    I still can't see how to use uniform continuity. Could you, please, explain it further?
    $endgroup$
    – lzralbu
    1 hour ago










  • $begingroup$
    I shall do so...
    $endgroup$
    – RRL
    1 hour ago










  • $begingroup$
    What about the example given in the question?
    $endgroup$
    – Jens Schwaiger
    46 mins ago










  • $begingroup$
    @JensSchwaiger: $cos(x^2)$ is not uniformly continuous on $[0,infty)$. OP introduced this as a counterexample for the L'Hospital trick. It is not relevant to the actual question where the assumption is that $f'$ is uniformly continuous.
    $endgroup$
    – RRL
    32 mins ago















$begingroup$
I can help you further, but first let me know if these hints makes it obvious to you now.
$endgroup$
– RRL
1 hour ago




$begingroup$
I can help you further, but first let me know if these hints makes it obvious to you now.
$endgroup$
– RRL
1 hour ago












$begingroup$
I still can't see how to use uniform continuity. Could you, please, explain it further?
$endgroup$
– lzralbu
1 hour ago




$begingroup$
I still can't see how to use uniform continuity. Could you, please, explain it further?
$endgroup$
– lzralbu
1 hour ago












$begingroup$
I shall do so...
$endgroup$
– RRL
1 hour ago




$begingroup$
I shall do so...
$endgroup$
– RRL
1 hour ago












$begingroup$
What about the example given in the question?
$endgroup$
– Jens Schwaiger
46 mins ago




$begingroup$
What about the example given in the question?
$endgroup$
– Jens Schwaiger
46 mins ago












$begingroup$
@JensSchwaiger: $cos(x^2)$ is not uniformly continuous on $[0,infty)$. OP introduced this as a counterexample for the L'Hospital trick. It is not relevant to the actual question where the assumption is that $f'$ is uniformly continuous.
$endgroup$
– RRL
32 mins ago




$begingroup$
@JensSchwaiger: $cos(x^2)$ is not uniformly continuous on $[0,infty)$. OP introduced this as a counterexample for the L'Hospital trick. It is not relevant to the actual question where the assumption is that $f'$ is uniformly continuous.
$endgroup$
– RRL
32 mins ago

















draft saved

draft discarded
















































Thanks for contributing an answer to Mathematics Stack Exchange!


  • Please be sure to answer the question. Provide details and share your research!

But avoid


  • Asking for help, clarification, or responding to other answers.

  • Making statements based on opinion; back them up with references or personal experience.

Use MathJax to format equations. MathJax reference.


To learn more, see our tips on writing great answers.




draft saved


draft discarded














StackExchange.ready(
function ()
StackExchange.openid.initPostLogin('.new-post-login', 'https%3a%2f%2fmath.stackexchange.com%2fquestions%2f3205125%2funiformly-continuous-derivative-implies-existence-of-limit%23new-answer', 'question_page');

);

Post as a guest















Required, but never shown





















































Required, but never shown














Required, but never shown












Required, but never shown







Required, but never shown

































Required, but never shown














Required, but never shown












Required, but never shown







Required, but never shown







Popular posts from this blog

Dapidodigma demeter Subspecies | Notae | Tabula navigationisDapidodigmaAfrotropical Butterflies: Lycaenidae - Subtribe IolainaAmplifica

Constantinus Vanšenkin Nexus externi | Tabula navigationisБольшая российская энциклопедияAmplifica

Vas sanguineum Index Historia | Divisio | Constructio anatomica | Vasorum sanguineorum morbi (angiopathiae) | Notae | Nexus interniTabula navigationisAmplifica